Anda di halaman 1dari 1

DEPARTMENT OF MATHEMATICS, IIT GUWAHATI

MA101: Mathematics I Quiz I (Maximum Marks: 10)


Date: August 26, 2011 Time: 8 am - 8:50 am
1. Let A be an n n matrix such that the system of equations Ax = 0 has a non-trivial solution. Is it
possible that the system of equations A
t
x = b has a unique solution for some b R
n
? Justify. 2
2. Examine whether the following sets are subspaces of R
2
: 1+1
(a) {[x, y]
t
: x R, y Z} (b) {[x, y]
t
R
2
: 2x +y = 0 or x + 2y = 0}.
3. Prove or disprove: If S = {v
1
, v
2
, v
3
, v
4
, v
5
, v
6
} is a linearly dependent set of vectors in R
5
then each
vector of S must be a linear combination of the remaining vectors of S. 2
4. Let A and B be two matrices of sizes m n and n m respectively and let m > n. Prove that the
matrix AB is not invertible. 2
5. Find a basis for the subspace spanned by the vectors [2, 1, 3, 1]
t
, [1, 2, 0, 1]
t
, [0, 2, 2, 1]
t
and
[1, 1, 1, 1]
t
. 2
End
DEPARTMENT OF MATHEMATICS, IIT GUWAHATI
MA101: Mathematics I Quiz I (Maximum Marks: 10)
Date: August 26, 2011 Time: 8 am - 8:50 am
1. Let A be an n n matrix such that the system of equations Ax = 0 has a non-trivial solution. Is it
possible that the system of equations A
t
x = b has a unique solution for some b R
n
? Justify. 2
2. Examine whether the following sets are subspaces of R
2
: 1+1
(a) {[x, y]
t
: x R, y Z} (b) {[x, y]
t
R
2
: 2x +y = 0 or x + 2y = 0}.
3. Prove or disprove: If S = {v
1
, v
2
, v
3
, v
4
, v
5
, v
6
} is a linearly dependent set of vectors in R
5
then each
vector of S must be a linear combination of the remaining vectors of S. 2
4. Let A and B be two matrices of sizes m n and n m respectively and let m > n. Prove that the
matrix AB is not invertible. 2
5. Find a basis for the subspace spanned by the vectors [2, 1, 3, 1]
t
, [1, 2, 0, 1]
t
, [0, 2, 2, 1]
t
and
[1, 1, 1, 1]
t
. 2
End

Anda mungkin juga menyukai